FORTINBERRY CH4 Flashcards

1
Q

During a home health visit, the physical therapist observed several items that require modification in the home of an elderly patient.
In terms of priority, the environmental hazard that needs the most immediate attention is
A. The cracked toilet seat
B. A malfunctioning thermostat
C. A throw rug
D. A cluttered kitchen

A

C.
The presence of a throw rug could result in a fall, which would be far more hazardous to the health of an elderlv client than the other obiects in the environment. In the elderly, falls are the major cause of fractures.

How well did you know this?
1
Not at all
2
3
4
5
Perfectly
2
Q

A 55-year-old patient sees a physical therapist for an examination of upper extremity function
1 week after Botox to the patient’s finger flexors in the right upper extremity. The patient had a stroke 1 year ago and is continuing to work on increasing function. During your examination you find that you are unable to fully extend the wrist and the fingers. One of the goals you establish with the patient is to increase ROM in this area. The best way to achieve this goal is by
A. AROM
B. PROM
C. Splinting to provide low load prolonged stretch
D. Stretching and weight bearing

A

C.
If soft tissue shortening has occurred over time, low load prolonged stretch splinting would be ideal. This technique is very different from stretching and weight bearing, which typically position the patient into the greatest amount of stretch tolerated. AROM and PROM will have significantly less effect on the structures.

How well did you know this?
1
Not at all
2
3
4
5
Perfectly
3
Q

A physical therapist is treating a patient with a Colles fracture. The patient’s forearm has been immobilized for 3 weeks and will require 4 additional weeks in the cast before the patient can begin functional tasks. An initial focus of treatment should be
A. Passive ROM (PROM)
B. Placement of the extremity in a sling
C. Movement of the joints surrounding the fracture
D. To avoid treatment until the cast is removed

A

C.
Fractured sites should remain stable to
promote healing and realignment of the bones.
However, the PT should encourage the movement of adjacent joints to assist in maintaining muscle strength and lengthening of tendons and muscles.

How well did you know this?
1
Not at all
2
3
4
5
Perfectly
4
Q

During a treatment session, the physical therapist observes that the patient can flex the affected shoulder through its full ROM in a side-lying position. The PT should progress to activities that place the extremitv in
A. A gravity-assisted position
B. A gravity-eliminated position
C. A neutral position
D. An antigravity position

A

D.
If grading shoulder flexion, the next step after achieving full shoulder flexion in a side-lying position is to begin to work or perform activities against gravity to begin increasing strength. Shoulder flexion against gravity is achieved with the individual in the sitting or standing position.

How well did you know this?
1
Not at all
2
3
4
5
Perfectly
5
Q

Which activity of daily living (ADL) activity would the PT caution a patient with a recent hip replacement to avoid?
A. Tying shoes
B. Pulling up pants
C. Putting on shirt
D. Bathing the back

A

A.
A person with a hip replacement should avoid any activity, such as shoe tying, which could potentially cause hip flexion to 90 degrees or greater. Such a position could actually undo the benefits of the surgical procedure.

How well did you know this?
1
Not at all
2
3
4
5
Perfectly
6
Q

During a treatment session, the PT simulates the need for the client to walk up stairs to a kitchen with a painful/weak leftleg. The patient should be instructed to move the
A. Leftleg up to the next step with the
cane
B. Rightleg up to the next step with the
cane
C. Rightleg up and then his left leg/cane
D. Leftleg up and then his right leg/cane

A

C.
When ascending or descending stairs, the cane should move with the painful/weak leg.
Specifically, when ascending the stairs the leg without the cane should move first, allowing the weak leg and cane to bear the weight for only a short amount of time until the strong leg is able to provide the needed stability.

How well did you know this?
1
Not at all
2
3
4
5
Perfectly
7
Q

You are working with a 53-year-old client who has had a right CVA. The patient is lying on a therapy mat, and you are performing passive
ROM to herleft arm. Once vou have the patient’s arm in 90 degrees of flexion, the patient complains of some discomfort and pain.
The best course of action would be to
A. Continue as tolerated, because passive
ROM must be maintained
B. Begin the ROM again and make sure the scapula is gliding
C. Continue and do not go past the point of pain
D. Consult an orthopedic specialist

A

B.
Discomfort and damage can occur if the scapula is not gliding with the humerus during movement. Passive ROM can cause damage if the structures are not moving properly. An orthopedic specialist may be beneficial if therapy interventions have not been successful.

How well did you know this?
1
Not at all
2
3
4
5
Perfectly
8
Q

The BEST strategy to use with a contracted joint that has a soft end field is to
A. Perform tendon gliding exercises
B. Applylow-load, long duration stretch
C. Use a quick stretch technique
D. Perform active ROM

A

B.
The term “soft end field” is a spongy quality at end range of a joint contracture. It usually indicates that the joint has the potential to remodel. A low-load, long duration stretch may yield the best results.

How well did you know this?
1
Not at all
2
3
4
5
Perfectly
9
Q

Which of the following is considered an absolute contraindication to manipulation?
A. Smoking and hypertension
B. Whiplash injury
C. Birth control pills and smoking
D. Acute myelopathy

A

D.
Acute myelopathy is considered an absolute contraindication to manipulation. This may be seen in cervical spondylotic myelopathy.
Smoking, hypertension, and use of birth control pills are considered risks for vertebrobasilar insufficiency.

How well did you know this?
1
Not at all
2
3
4
5
Perfectly
10
Q

Which of the following disc herniations would you expect to respond MOST favorably to traction therapy?
A. Medial to the nerve root
B. Lateralto the nerve root
C. Anterior to the nerve root
D. Posterior to the nerve root

A

B.
When the disc is in the axilla of the nerve root (medial), axial traction may irritate the problem.

How well did you know this?
1
Not at all
2
3
4
5
Perfectly
11
Q

For the best protection of lumbar mechanics, the driver’s car seat should be positioned
A. As far from the steering wheel as possible
B. With the front of the seat lower than the back of the seat
C. With the entire seat bottom level with the floor of the car
D. As close to the steering wheel as practical

A

D.
With the seat close to the pedals, the lumbopelvic region is flexed, separating the posterior facets and disc space at L5-S1. Adding a lumbar pillow supports the lumbar curve at the same time.

How well did you know this?
1
Not at all
2
3
4
5
Perfectly
12
Q

A pitcher is exercising in a clinic with a sports cord mounted behind and above his head. The pitcher simulates the pitching motion using the sports cord as resistance. Which proprioceptive neuromuscular facilitation (PNF) diagonal is the pitcher using to strengthen the muscles involved in pitching a baseball?
A. D1 extension
B. D1 flexion
C. D2 extension
D. D2 flexion

A

C.
The pitcher is moving into D2 extension with the throwing motion. He is strengthening the muscles involved in shoulder internal rotation, adduction, and forearm pronation.

How well did you know this?
1
Not at all
2
3
4
5
Perfectly
13
Q

A therapist is mobilizing a patient’s right shoulder. The movement taking place at the joint capsule is not completely to end range. Itis a large-amplitude movement from near the beginning of available range to near the end of available range. What grade mobilization, according to Maitland, is being performed?
A. Grade I
B. Grade II
C. Grade III
D. Grade IV

A

B.
Grade is a small oscillating movement at the beginning of range. Grade Ill is a large movement up to the end of available range.
Grade IV is a small movement at the end of available range.

How well did you know this?
1
Not at all
2
3
4
5
Perfectly
14
Q

A 29-year-old woman is referred to a therapist with a diagnosis of recurrent ankle sprains. The patient has a history of several inversion ankle sprains within the past year. No edema or redness is noted at this time. Which of the following is the best treatment plan?
A. Gastrocnemius stretching, ankle strengthening, andice
B. Rest, ice, compression, elevation, and ankle strengthening
C. Ankle strengthening and a proprioception program
D. Rest. ice, compression, elevation, and gastrocnemius stretching

A

C.
Patients with recurrent ankle sprains benefit from proprioceptive exercises. Choices B and Dare not indicated because of the lack of acute signs and symptoms. Choice A is a good plan, but not the most correct because there is no mention of proprioception.

How well did you know this?
1
Not at all
2
3
4
5
Perfectly
15
Q

The therapist is treating a male patient for a second-degree acromioclavicular sprain. The patient has just finished the doctor’s prescription of 3 sessions/week for 4 weeks.
The therapist is treating the patient with iontophoresis (driving dexamethasone), deltoid-strengthening exercises, pectoral-strengthening exercises, and ice. The patient reports no decline in pain level since the initial examination. Which of the following is the best course of action for the therapist?
A. Phone the doctor and request continued phvsical therapv.
B. Tell the patient to go back to the doctor because he is not making appropriate progress.
C. Discharge the patient because he will improve on his own.
D. Take the problem to the supervisor of the facility.

A

B.
The patient should have made adequate progress in this period with this protocol.
Because of the lack of progress, the patient needs further evaluation by the physician.

How well did you know this?
1
Not at all
2
3
4
5
Perfectly
16
Q

A therapist working in an outpatient physical therapy clinic examines a patient with a diagnosis of rotator cuff bursitis. The physician’s orderis to examine and treat.
During the examination the following facts are revealed:
* Active shoulder flexion = 85 degrees
with pain;
* Passive shoulder flexion = 177 degrees;
* Active shoulder abduction = 93 degrees
with pain;
* Passive shoulder abduction = 181 degrees;
* Active external rotation = 13 degrees
with pain;
* Passive eternal rotation = 87 degrees;
* Drop arm test = positive;
* Impingement test = negative;
* Biceps tendon subluxation test = negative;
* Sulcus sign = negative.
Of the following, which is the best course of action?
A. Treat the patient for 1 week with moist heat application, joint mobilization, and strengthening, Then suggest to the patient that he or she return to the physician if there are no positive results.
B. Treat the patient for 1 week with ultrasound, strengthening, and ice.
Then suggest to the patient that he or she return to the physician if there are no positive results.
C. Treat the patient for 1 week with a home exercise program, strengthening, passive range of motion by the therapist, and ice. Then suggest to the patient that he or she return to the physician if there are no positive results.
D. Treat the patient for 1 week with strengthening, a home exercise program, and ice. Then suggest to the patient that he or she return to the phvsician if there are no positive results.

A

D.
The patient most likely has a rotator cuff tear. Choices A and B are incorrect because there is no need for heating modalities. Choice C is wrong because the patient has full passive range of motion.

How well did you know this?
1
Not at all
2
3
4
5
Perfectly
17
Q

The therapist is crutch training a 26-year-old man who underwent right knee arthroscopy 10 hours ago. The patient’s weight-bearing status is toe-touch weight-bearing on the right lower extremity. If the patient is going up steps, which of the following is the correct sequence of verbal instructions?
A. “Have someone stand below you while going up, bring the left leg up first, then the crutches and the right leg.
B.”Have someone stand above you while going up, bring the left leg up first, then the crutches and the right leg.”
C.”Have someone stand below you while going up, bring the right leg up first, then the crutches and the left leg.”
D.”Have someone stand above you while going up, bring the right leg up first, then the crutches and the right leg.”

A

A.
Choice A is the correct gait sequence for ascending stairs in the given scenario. A caregiver should stand below the patient because the patient is most likelv to fall down the stairs. This same rule holds true for descending stairs.

How well did you know this?
1
Not at all
2
3
4
5
Perfectly
18
Q

What is the best way to first exercise the postural (or extensor) musculature when it is extremelyweak to facilitate muscle control?
A. Isometrically
B. Concentrically
C. Eccentrically
D. Isokinetically

A

A.
Isometric exercises in the shortest range of the extensor muscle are used to begin strengthening. In contrast, weak flexor muscles should be strengthened in the middle-to-lengthened range, because they most often work near their end range.

How well did you know this?
1
Not at all
2
3
4
5
Perfectly
19
Q

A 42-year-old receptionist presents to an outpatient physical therapy clinic complaining of low back pain. The therapist decides that postural modification needs to be part of the
treatment plan. What is the best position for the lower extremities while the patient is sitting?
A. 90 degrees of hip flexion, 90 degrees of knee flexion, and 10 degrees of dorsiflexion
B. 60 degrees of hip flexion, 90 degrees of knee flexion, and 0 degrees of dorsiflexion
C. 110 degrees of hip flexion, 80 degrees
of knee flexion, and 10 degrees of dorsiflexion
D. 90 degrees of hip flexion, 90 degrees of knee flexion, and 0 degrees of dorsiflexion

A

D.
This position places the least amount of stress on the lumbar spine in the sitting position.

How well did you know this?
1
Not at all
2
3
4
5
Perfectly
20
Q

A 67-year-old woman presents to an outpatient facility with a diagnosis of right adhesive capsulitis. The therapist plans to focus mostly on gaining abduction range of motion. In which direction should the therapist mobilize the shoulder to gain abduction range of motion?
A. Posteriorly
B. Anteriorly
C. Inferiorly
D. Superiorly

A

C.
The therapist must stretch the inferior portion of the capsule in an effort to gain abduction of the involved shoulder. This principle is supported by the convex-concave rule.

How well did you know this?
1
Not at all
2
3
4
5
Perfectly
21
Q

A patient is positioned in the supine position.
The involved left upper extremity is positioned by the therapist in 90 degrees of shoulder flexion. The therapist applies resistance into shoulder flexion, then extension. No movement takes place. The therapist instructs the patient to “hold” when resistance is applied in both directions. Which of the following proprioceptive neuromuscular facilitation techniques is being used?
A. Repeated contractions
B. Hold-relax
C. Rhythmic stabilization
D. Contract-relax

A

C.
Rhvthmic stabilization involves a series
of isometric contractions of the agonist then the antagonist.

How well did you know this?
1
Not at all
2
3
4
5
Perfectly
22
Q

The therapist is treating a patient who recently received a below-knee amputation. The therapist notices in the patient’s chart that a psychiatrist has stated that the patient is in the second stage of the grieving process. Which stage of the grieving process is this patient most likely exhibiting?
A. Denial
B. Acceptance
C. Depression
D. Anger

A

D.
The five stages of grieving are (in order from first to last) denial, anger, bargaining, depression, and acceptance.

How well did you know this?
1
Not at all
2
3
4
5
Perfectly
23
Q

A 32-year-old man is referred to physical therapy with the diagnosis of a recent complete anterior cruciate ligament tear. The patient and the physician have decided to avoid surgery as long as possible. The therapist provides the patient with a home exercise program and instructions about activities that will be limited secondary to this diagnosis. Which of the following is the best advice?
A. There are no precautions.
B. The patient should avoid all athletic activity for 1 year.
C. The patient should avoid all athletic activity until there is a minimum of 20difference in the bilateral quadriceps muscle as measured isokinetically.
D. The patient should wear abrace and compete in only light athletic events.

A

D.
The ACL-deficient patient has a significant rotatory instability. Bracing may prevent some of this instability. Sports that are especially difticult on the knees (e.g., skiing, competitive tennis) are contraindicated.

How well did you know this?
1
Not at all
2
3
4
5
Perfectly
24
Q

A physician has ordered a physical therapist to treat a patient with chronic low back pain. The order is to “increase gluteal muscle function by decreasing trigger points in the quadratuslumborum.” What is the first technique that should be used by the physical therapist?
A. Isometric gluteal strengthening
B. Posture program
C. Soft tissue massage
D. Muscle reeducation

A

C.
Trigger points are often treated with soft tissue massage. Othertechniques include strain/counterstrain, mvofascial release, and muscle energy techniques.

How well did you know this?
1
Not at all
2
3
4
5
Perfectly
25
Q

A 60-year-old woman is referred to outpatient physical therapy services for rehabilitation after receiving a left total knee replacement 4 weeks ago. The patient is currently ambulating with a standard walker with a severely antalgic gait pattern. Before the recent surgery the patient was ambulating independently without an assistive device. Left knee flexion was measured in the initial examination and found to be 85 degrees actively and 94 degrees passively. The patient also lacked 10 degrees of full passive extension and 17 degrees of full active extension. Which of the following does the therapist need to first address?
A. Lack of passive left knee flexion
B. Lack of passive left knee extension
C. Lack of active left knee extension
D. Ability to ambulate with a lesser assistive device

A

B.
Choice B is correct because the patient has to achieve passive knee extension before she can gain full active knee extension. Full active knee extension and full flexion are important and should be a major focus of the patient’s session, but the question asks for the most serious deficit. Ambulating with a lesser assistive device should be the focus at a later time because the patient’s gait is still severely antalgic and obvious instability is still present.
Usually a patient is advanced to a lesser assistive device when he or she can ambulate without large gait deviations with the current assistive device.

How well did you know this?
1
Not at all
2
3
4
5
Perfectly
26
Q

A home health physical therapist is sentto examine a 56-year-old man who has suffered a recent stroke. The patient is sitting in a lift chair, accompanied by his 14-year-old nephew. He seems confused several times throughout the examination. The nephew is unable to assist in clarifying much of the subiective history. The patient reports to the therapist that he is independent in ambulation with a standard walker as an assistive device and in all transfers without an assistive device. Based on the above information, which of the following sequence of events, chosen by the therapist, is in the correct order?
A. Ambulate with the standard walker with the wheelchair in close proximity;
transfer sit to stand in front of the wheelchair; transfer wheelchair to bed: assess range of motion and strength of all extremities in supine position
B. Ambulate with the standard walker with the wheelchair in close proximity; transfer wheelchair to bed; assess range of motion and strength of all extremities in supine position; transfer sit to stand at bedside
C. Assess range of motion and strength of all extremities in the lift chair; transfer sit to stand in front of the lift chair; ambulate with the standard walker with the wheelchair in close proximity; transfer wheelchair to bed
D. Assess range of motion and strength of all extremities in the lift chair; ambulate with the standard walker with the wheelchair in close proximity; transfer sit to stand in front of the wheelchair; transfer wheelchair to bed

A

C.
Because of the unreliable history
obtained in the evaluation, the therapist at least should make a quick assessment of range of motion and strength before the patient attempts to stand. Sit-to-stand transfer should then be assessed in front of the lift chair before the patient attempts to ambulate.

How well did you know this?
1
Not at all
2
3
4
5
Perfectly
27
Q

A patient is receiving crutch training 1 day after a right knee arthroscopic surgery. The patient’s weight-bearing status is toe-touch weight-bearing on the right lower extremity. The therapist first chooses to instruct the patient how to perform a correct sit-to-stand transfer.
Which of the following is the most correct set of instructions?
A. (1) Slide forward to the edge of the chair;
(2) put both the crutches in front of you and hold both grips together with the right hand; (3) press on the left arm rest with the left hand and the grips with the right hand: (4) lean forward;
(5) stand up, placing your weight on the left lower extremity; (6) place one crutch slowlv under the left arm, then under the right arm.
B. (1) Slide forward; (2) put one crutch in each hand, holding the grips; (3) place crutches in a vertical position; (4) press down on the grips; (5) stand up, placing more weight on the left lower extremity.
C. (1) Slide forward to the edge of the chair;
(2) put both the crutches in front of vou and hold both grips together with the left hand: (3) press on the right arm rest with the right hand and the grips with the left hand; (4) lean forward; (5) stand up, placing your weight on the left lower extremity: (6) place one crutch slowly under the right arm, then under the left arm.
D. (1) Place crutches in close proximity; (2) slide forward; (3) place hands on the arm rests; (4) press down and stand up;
(5) place weight on the left lower extremity; (6) reach slowly for the crutches and place under the axilla.

A

A.
The method used in Choice A is the safest. The method used in Choice C is too unstable.

How well did you know this?
1
Not at all
2
3
4
5
Perfectly
28
Q

A 20-year-old man with anterior cruciate ligament reconstruction with allograft presents to an outpatient physical therapy clinic. The patient’s surgerywas 5 days ago. The patient is independent in ambulation with crutches. He also currently has 53 degrees of active knee flexion and 67 degrees of passive knee flexion andlacks 10 degrees of full knee extension actively and 5 degrees passively. What is the most significant deficit on which the physical therapist should focus treatment?
A. Lack of active knee extension
B. Lack of passive knee extension
C. Lack of active knee flexion
D. Lack of passive knee flexion

A

B.
Passive extension is the most important motion to gain after an anterior cruciate ligament reconstruction, regardless of the graft tvpe. Active extension can be achieved once passive extension is full (or equal bilaterally).

How well did you know this?
1
Not at all
2
3
4
5
Perfectly
29
Q

A physical therapist is ordered to examine and treat in the acute setting a patient who received a left total knee replacement 1 day ago. Before surgery, the patient was independent in all activities of daily living, transfers, and ambulation with an assistive device. The family reports that ambulation was slow and guarded because of knee pain. The physician’s orders are to ambulate with partial weight bearing on the left lower extremity and to increase strength/range of motion. At this point, bed-to-wheelchair transfers, sit-to-stand transfers, and wheelchair-to-toilet transfers require the minimal assistance of one person. The left knee has 63 degrees of active flexion and 77 degrees of passive flexion. The left knee also lacks 7 degrees of full extension actively and 3 degrees passively. Right hip strength is recorded as
follows: hip flexion and abduction = 4/5, hip
adduction and extension = 5/5, knee flexion =
4/5, knee extension = 5/5, ankle plantar flexion
= 4/5. and dorsiflexion = 5/5. Leftlower
extremity strength is recorded as follows: hip
flexion = 3/5, hip abduction and adduction =
3/5, hip extension =3/5, knee flexion and
extension = 3-/5, ankle dorsiflexion = 3/5, and
plantar flexion = 3/5. The patient is currently
able to ambulate 30 feet x 2 with a standard walker and minimal assistance of one person on level surfaces. She also ambulates with a flexed knee throughout the gait cycle. According to the physician, she most likely will be discharged home (with home health services), where she lives alone, within the next 2 to 3 days. Which is the most important long-term goal in the acute setting?
A. In 3 davs the patient will be independent in all transfers.
B. In 3 days the patient will ambulate with a quad cane independently, with no gait deviations, on level surfaces 50 feet x 3.
C. In 3 days the patient will increase all left lower extremity manual muscle testing grades by one half grade.
D. In 3 days the patient will have active left knee range of motion from 0 to 90 degrees and passive range of motion from 0 to 95 degrees.

A

A.
Because the patient lives alone,
independent transfer is the most important goal listed. Functional ambulation is an important goal, but choice B is an unrealistic goal for the patient to accomplish in a 2- or 3-day period.

How well did you know this?
1
Not at all
2
3
4
5
Perfectly
30
Q

A patient presents to therapy with poor motor control of the lower extremities. The therapist determines that to work efficiently toward the goal of returning the patient to his prior level of ambulation, he must work in the following order regarding stages of control
A. Mobility, controlled mobility, stability, skill
B. Stability, controlled stability, mobility,
skill
C. Skill, controlled stability, controlled mobility
D. Mobility, stability, controlled mobility, skill

A

D.
This choice lists the stages of control in the correct order.

How well did you know this?
1
Not at all
2
3
4
5
Perfectly
31
Q

A 23-year-old woman arrives at an outpatient physical therapy clinic with a prescription to examine and treat the right hand. One week earlier the patient underwent surgical repair of the flexor tendons of the right hand at zone 2.
She also had her cast removed at the physician’s office a few minutes before coming to physical therapy. What is the best course of treatment for this patient?
A. Ultrasound to decrease scarring
B. Gentle grip strengthening with putty
C. Splinting the distal interphalangeal joint and proximal interphalangeal joints at neutral
D. Splinting with the use of rubber bands to passively flexthe fingers

A

D.
Choice D is the correct treatment.
Strengthening is not indicated at this time, and splinting as described in Choice C places too much stretch on the tendons. In addition, static splinting does not allow tendon gliding.
Ultrasound is contraindicated over a healing tendon repair.

How well did you know this?
1
Not at all
2
3
4
5
Perfectly
32
Q

A 67-year-old man with a below-knee amputation presents to an outpatient clinic. His surgical amputation was 3 weeks ago, and his scars are well healed. Which of the following is incorrect information about stump care?
A. Use a light lotion on the stump after bathing each night.
B. Continue with use of a shrinker 12 hours per day.
C. Wash the stump with mild soap and water.
D. Use scar massage techniques.

A

B.
The shrinker should be removed only
for bathing. Because the surgical scars are healed, the stump can be immersed in water.

How well did you know this?
1
Not at all
2
3
4
5
Perfectly
33
Q

A physical therapist is teaching a class in geriatric fitness/strengthening at a local gym.
Which of the following is not a general guideline for exercise prescription in this patient population?
A. To increase exercise intensity, increase treadmill speed rather than the grade.
B. Start at a low intensity (2 to 3 METs).
C. Use machines for strength training rather than free weights.
D. Set weight resistance so that the patient can perform more than 8 repetitions before fatigue.

A

A.
Because of poor balance, geriatric patients should increase the treadmill grade rather than the speed. Use of machines allows better posture and low intensities and limits the exercise within the patient’s safe range of motion.

How well did you know this?
1
Not at all
2
3
4
5
Perfectly
34
Q

A 76-year-old woman received a cemented right total hip arthroplasty (THA) 24 hours ago. The surgeon documented that he used a posterolateral incision. Which of the following suggestions is inappropriate for the next 24 hours?
A. Avoid hip flexion above 30 degrees.
B. Avoid hip adduction past midline.
C. Avoid any internal rotation.
D. Avoid abduction past 15 degrees.

A

D.
Movements that stress the
posterolateral hip joint capsule should be avoided. Sources vary on the exact amount of flexion that should be avoided. Passive hip abduction should be maintained after surgery with a wedge.

How well did you know this?
1
Not at all
2
3
4
5
Perfectly
35
Q

The therapist is examining a 38-year-old man who complains of right sacroiliac joint pain. The therapist decides to assess leglength discrepancy in supine versus sitting position.
When the patient is in supine position, leg lengths are equal; however, when the patient rises to the sitting position, the right lower extremity appears 2 cm shorter. Which of the following should be a part of the treatment plan?
A. Right posterior SI mobilization
B. Right anterior SI mobilization
C. Left posterior SI mobilization
D. Left anterior SI mobilization

A

A.
This patient most likely has a right anterior rotation of the right innominate and thus needs right posterior mobilization of the right innominate.

How well did you know this?
1
Not at all
2
3
4
5
Perfectly
36
Q

Intaping an athlete’s ankle prophylactically before a football game, in what position should the ankle be slightly positioned before taping to provide the most protection against an ankle sprain?
A. Inversion, dorsiflexion, abduction
B. Eversion, plantar flexion, adduction
C. Eversion, dorsiflexion, abduction
D. Inversion, plantar flexion, adduction

A

C.
This position, which limits inversion, plantar flexion, and adduction, is the most common position for ankle sprains.

How well did you know this?
1
Not at all
2
3
4
5
Perfectly
37
Q

A physical therapist is treating a 35-year-old man with traumatic injury to the right hand.
The patient has several surgical scars from a tendon repair performed 6 weeks ago. What is the appropriate type of massage for the patient’s scars?
A. Massage should be transverse and longitudinal.
B. Massage should be circular and longitudinal.
C. Massage should be transverse and circular.
D. Massage is contraindicated after a tendon repair.

A

C.
Transverse (perpendicular to the scar) or circular massage assists in mobilization of scar tissue.

How well did you know this?
1
Not at all
2
3
4
5
Perfectly
38
Q

A patient is being treated in an outpatient facility after receiving a meniscus repair to the right knee 1 week ago. The patient has full passive extension of the involved knee but lacks 4 degrees of full extension when performing a straight leg raise. The patient’s active flexion is 110 degrees and passive flexion is 119 degrees.
What is a common term used to describe the patient’s most significant range of motion deficit? What is a possible source of this problem?
A. Flexion contracture, quadricep atrophy
B. Extension lag, joint effusion
C. Flexion lag, weak quadriceps
D. Extension contracture, tight hamstrings

A

B.
The patient has an extension lag, which may be due to any source that has inhibited the quadriceps and results in an inability to fully extend the knee actively.

How well did you know this?
1
Not at all
2
3
4
5
Perfectly
39
Q

A physical therapist is attempting to increase a patients functional mobility in a seated position.
To treat the patient most effectively and efficiently, the following should be performed in what order?
1. Weight shifting of the pelvis
2. Isometric contractions of the lower extremity
3. Trunk range of motion exercises
4. Isotonic resistance to the quadriceps
A. 1, 2,3,4
B. 2,3,1,4
C. 4, 3.2, 1
D. 3, 2, 1, 4

A

D.
The treatment techniques should be performed in the order of mobility, stability, controlled mobility, and skill.

How well did you know this?
1
Not at all
2
3
4
5
Perfectly
40
Q

A physical therapist is speaking to a group of avid tennis players. The group asks how to prevent tennis elbow (lateral epicondylitis).
Which of the following is incorrect information?
A. Primarily use the wrist and elbow extensors during a backhand stroke.
B. Begin the backhand stroke in shoulder adduction and internal rotation.
C. Use a racket that has a large grip.
D. Use a light racket.

A

A.
Tennis elbow results from overuse of
the wrist extensors. The shoulder external rotators should be used to power a backhand.

How well did you know this?
1
Not at all
2
3
4
5
Perfectly
41
Q

A physical therapist is fabricating a splint for a patient who received four metacarpophalangeal joint replacements. The surgical joint replacement was necessarv because of severe rheumatoid arthritis. Which of the following is the correct placement of the metacarpophalangeal joints in the splint?
A. Full flexion and slight radial pull
B. Full flexion and slight ulnar pull
C. Full extension and slight radial pull
D. Full extension and slight ulnar pull

A

C.
The radial pull component is designed to allow tightening of the radial side of the capsule.

How well did you know this?
1
Not at all
2
3
4
5
Perfectly
42
Q

A therapist is ordered to fabricate a splint for a
2-month-old infant with congenital hip dislocation. In what position should the hip be placed while in the splint?
A. Flexion and adduction
B. Extension and adduction
C. Extension and abduction
D. Flexion and abduction

A

D.
This is the most stable position of the
hip, which allows for more normal growth.

How well did you know this?
1
Not at all
2
3
4
5
Perfectly
43
Q

A physical therapist is discharging a 32-year-old man from outpatient physical therapy. The patient received therapy for a traumatic ankle injury that occurred several months earlier. The surgery performed on the patient’s ankle required placement of plates and screws, which resulted in a permanent range of motion deficit of 10 degrees of active and passive dorsiflexion.
Strength in the ankle is 5/5 with manual muscle testing. Of the following, which is the highest functional outcome that the patient can expect?
A. Independent ambulation with no gait deviations
B. Ambulation with a cane with minimal gait deviations
C. Running with no gait deviations
D. Ascending or descending stairs with no gait deviations

A

A.
Normally, the ankle requires 20 degrees or more of dorsiflexion for a patient to run or ascend/descend stairs properly. Independent ambulation with a normal gait pattern requires 10 degrees of dorsiflexion.

How well did you know this?
1
Not at all
2
3
4
5
Perfectly
44
Q

A physical therapist is performing passive range of motion on the shoulder of a 43-year-old woman who received a rotator cuff repair 5 weeks ago. During passive range of motion, the therapist notes a capsular end feel at 95 degrees of shoulder flexion. What should the therapist do?
A. Begin isokinetic exercise at 180 degrees
per second.
B. Begin joint mobilization.
C. Schedule the patient an appointment with the physician immediately.
D. Begin aggressive supraspinatus activity.

A

B.
Because of the length of the time since
the surgical procedure, the patient may have adhesive capsulitis. The capsule should continue to be stretched to increase range of motion. The patient should visit the physician if the range of motion deficits continue. Active exercise may be necessary at this stage of recovery, but it will not help to relieve a capsular dysfunction.

How well did you know this?
1
Not at all
2
3
4
5
Perfectly
45
Q

In which of the following situations should the therapist be most concerned about the complications resulting from grade IV joint mobilization techniques?
A. A 37-year-old man with a Colles fracture suffered 10 weeks ago
B. A 23-year-old woman with a boxer’s fracture suffered 10 weeks ago
C. A 34-year-old man with a scaphoid fracture suffered 12 weeks ago
D. A 53-year-manwith a Bennett’s fracture suffered 12 weeks ago

A

C.
Because the scaphoid has a poor vascular supply, aggressive therapy should be avoided until the bone is fully healed (12 to 24 weeks). A Colles fracture (fracture of the distal radius with dorsal movement of the fixed segment) should heal in 6 to 8 weeks. A boxer’s fracture (fracture of the fifth metacarpal) requires 4 to 6 weeks. A Bennett’s fracture (fracture of the proximal first metacarpal) usually requires 6 to 8 weeks. The length of healing time given in the above examples obviously depends on the individual patient and the type of surgical fixation (if any).

How well did you know this?
1
Not at all
2
3
4
5
Perfectly
46
Q

Which of the following is an inappropriate exercise for a patient who received an anterior cruciate ligament reconstruction with a patella tendon autograft 2 weeks ago?
A. Lateral step-ups
B. Heelslides
C. Stationary bike
D. Pool walking

A

A.
Lateral step-ups are probably too difficult for a patient who received an anterior ligament reconstruction with a patella tendon autograft 2 weeks ago.

How well did you know this?
1
Not at all
2
3
4
5
Perfectly
47
Q

A physical therapist is speaking to a group of receptionists about correct posture. Which of the following is incorrect information?
A. Position computer monitors at eye level.
B. Position seats so that the feet are flat on the floor while sitting.
C. Position kevboards so that the wrists are in approximately 20 degrees of extension.
D. Take frequent stretching breaks.

A

C.
The wrists should be in neutral position when the fingers are on the middle row of the keyboard.

How well did you know this?
1
Not at all
2
3
4
5
Perfectly
48
Q

A physical therapist is treating an automobile mechanic. The patient asks for tips on preventing upper extremity repetitive motion injuries. Which of the following is incorrect advice?
A. Use vour entire hand rather than just the fingers when holding an object.
B. Position tasks so that they are performed below shoulder height.
C. Use tools with small straight handles when possible.
D. When performing a forceful task. keep the materials slightly lower than the elbow.

A

C.
Tools with small handles require more grip strength. Tasks below shoulder height reduce the risk of impingement, and more force can be applied to tasks if they are kept below elbow height.

How well did you know this?
1
Not at all
2
3
4
5
Perfectly
49
Q

A patient presents to physical therapy with a long-standing diagnosis of bilateral pesplanus.
The therapist has given the patient custom-fit orthotics. After using the orthotics for 1 week, the patient complains of pain along the first metatarsal. The therapist decides to use joint mobilization techniques to decrease the patient’s pain. In which direction should the therapist mobilize the first metatarsal?
A. Inferiorly
B. Superiorly
C. Laterally
D. Medially

A

A.
In response to a pronated subtalar joint, the forefoot undergoes a supination twist and the first ray dorsiflexes. Because the distal first cuneiform is convex and the proximal first metatarsal is concave, inferior mobilization of the first metatarsal is required.

How well did you know this?
1
Not at all
2
3
4
5
Perfectly
50
Q

A 14-year-old boy with a diagnosis of osteosarcoma of his right distal femur underwent resection of the distal third of his femur and implantation of an expandable endoprosthetic device 2 months ago. He is now referred to outpatient physical therapy with no restrictions except PWB gait with crutches.
What impairment would you expect to most interfere with function at the time of the examination?
A. Leglength discrepancy
B. Limited right knee ROM
C. Limited righthip ROM
D. Pain at the site of surgical intervention

A

B. The leg length discrepancy would have been resolved by the surgical procedure, and pain at the surgical site will have diminished by 2 months postoperatively. Knee ROM will be more limited than hip because the procedure involved the distal femur.

How well did you know this?
1
Not at all
2
3
4
5
Perfectly
51
Q

A 4-year-old child diagnosed with osteosarcoma of the distal femur, is scheduled for resection of the distal third of the femur. What surgical intervention would provide the best long term functional outcome?
A. Allograft
B. Endoprosthetic implant
C. Hip disarticulation
D. Rotationplasty

A

D.
An endoprosthetic implant is limited in
the amount of growth that can be
accommodated. Allografts are only appropriate for children nearing skeletal maturity, and a hip disarticulation would not allow for normal active play even with a prosthesis.
Rotationplasty is a radical surgery, but it is the best option in this case.

How well did you know this?
1
Not at all
2
3
4
5
Perfectly
52
Q

A 6-month-old infant with acetabular dysplasia of the right hip diagnosed by radiograph, with a history of a dislocatable hip at birth, would usually be treated with
A. Arthrogram and closed reduction
B. Spica cast
C. Pavlik harness
D. Open reduction

A

C.
The Pavlik harness allows the hips to be maintained in flexion and abduction by limiting extension and adduction. This position limits avascular necrosis common with this diagnosis.

How well did you know this?
1
Not at all
2
3
4
5
Perfectly
53
Q

Which degree of strain in the following joints would normally take the longest amount of time to rehabilitate?
A. Grade I medial collateral ligament of the knee injury
B. Grade I anterior cruciate ligament iniurv
C. Grade Il ulnar collateralligament of the elbow injurv
D. Grade IlI anterior talofibular ligament injury

A

D.
Grade IlI injuries are complete ruptures
of the ligament involved. Grade I injuries are considered minor, while grade Il injuries will have associated edema, pain, and some loss of joint stability.

How well did you know this?
1
Not at all
2
3
4
5
Perfectly
54
Q

Its 6 weeks after acromioplasty and a patient is showing difficulty performing shoulder flexion and scaption exercises correctly. The patient shows shoulder “hike” above 70 degrees of shoulder flexion. Which of the following interventions would most quickly improve this problem?
A. Eccentric elbow flexion
B. Heavy resistance supraspinatus exercise
C. Gravity resistance supraspinatus exercise
D. Uppertrapezius strengthening

A

C.
Upper trapezius strengthening will only exacerbate this dysfunction, and the elbow exercise is irrelevant to this type of biomechanical problem. The supraspinatus responds best to gravity-resisted exercise early and a slow progression of resistance not to exceed 3 to 5 pounds.

How well did you know this?
1
Not at all
2
3
4
5
Perfectly
55
Q

Shoulder ROM is restricted in a patient 8 weeks after rotator cuff repair. Internal rotation and horizontal adduction are the most restricted motions. Which portion of the shoulder capsule should be stretched or mobilized?
A. Anterior
B. Posterior
C. Inferior
D. Superior

A

B.
The arthrokinematics of the shoulder
joint would lead one to believe that the posterior capsule is the most in need of mobilization.

How well did you know this?
1
Not at all
2
3
4
5
Perfectly
56
Q

A patient who underwent an acromioplasty 8 weeks ago presents with complaints of pain when reaching overhead and during the last 30 degrees of shoulder flexion. End range pain is also felt when using PROM into horizontal adduction, shoulder flexion, and shoulder abduction. Which of the following treatments would be most helpful for this patient?
A. Shoulder mobilizations for the anterior shoulder capsule
B. Shoulder mobilizations for the superior shoulder capsule
C. Acromioclavicular joint mobilization with the upper extremity in 20 degrees of shoulder flexion
D. Acromioclavicular joint mobilization with the upper extremity in 140 degrees of shoulder flexion

A

D. Since the motion restriction occurs in the upper ranges of flexion, mobilizations should focus on this portion of ROM. Arguments could be made for glenohumeral mobilization for the posterior and inferior capsule, but those choices do not exist.

How well did you know this?
1
Not at all
2
3
4
5
Perfectly
57
Q

A patient who underwent shoulder acromioplasty 6 days ago presents with pain and limited use for the involved upper extremity during ADLs. What is the most appropriate advice to decrease this patient’s pain while at home?
A. Discontinue use of sling and ice at home.
B. Use a sling during waking hours and ice throughout the day.
C. Begin progressive resistance exercises at home.
D. Discontinue use of a sling and use a moist heat pad at home.

A

B.
The shoulder is still early in rehabilitation at 6 days postoperatively.
Protection by the sling (along with ice for pain control) is a good suggestion. It is too early for aggressive exercise, and heat should never be used at this stage of recovery.

How well did you know this?
1
Not at all
2
3
4
5
Perfectly
58
Q

Considering a patient with recent anterior capsulolabral reconstruction, when can active range of motion (AROM) of the shoulder be initiated?
A. As soon as 1 to 2 days after surgery
B. 2 to 3 weeks postoperatively
C. 4 to 6 weeks postoperatively
D. 6 to 8 weeks postoperatively

A

A.
Since no trauma to the shoulder musculature is involved with this procedure, AROM can begin immediately (within painful limits and surgical guidelines).

How well did you know this?
1
Not at all
2
3
4
5
Perfectly
59
Q

In an outpatient physical therapy clinic, a patient presents with complaints of pain with elbow flexion at the anterior shoulder. He underwent anterior capsulolabral
reconstruction 10 weeks ago. Shoulder ROM is restricted in internal rotation, but all other motions are normal. Elbow ROM is normal. but painful at 90 to 100 degrees of elbow flexion.
What is the most appropriate course of action by the physical therapist?
A. Shoulder posterior mobilization, and treatment for biceps tendonitis
B. Shoulder anterior mobilization, and treatment for biceps tendonitis
C. Shoulder posterior mobilization only
D. Shoulder anterior mobilization only

A

A.
Posterior mobilization would release any restriction on ROM into internal rotation.
Lack of treatment for the biceps tendonitis symptoms would possibly delay strengthening of the upper extremity.

How well did you know this?
1
Not at all
2
3
4
5
Perfectly
60
Q

During an intervention session, a patient with recent (1 week ago) rotator cuff repair complains of cervical pain. His complaints are in the upper trapezius and medial scapular area of the involved upper extremity. What is the most appropriate course of action by the physical therapist?
A. Applyice to the area of complaint.
B. Assure the patient this is normal and continue with PROM treatments.
C. Call the physician immediately.
D. Examine the cervical spine.

A

D.
Although these complaints are very common after this particular procedure, the cervical spine should be examined. A cervical condition may have been masked by shoulder pain. Often there is abnormal muscle tone in this area as a response to the surgery.

How well did you know this?
1
Not at all
2
3
4
5
Perfectly
61
Q

A baseball pitcher underwent rotator cuff repair
8 weeks ago. Which portion of the shoulder capsule does not need to be mobilized under normal conditions?
A. Anterior
B. Posterior
C. Superior
D. Inferior

A

A.
The anterior capsule in the overhead throwing athlete should not be mobilized or stretched. Typically these athletes already have hypermobilty in this area. Treatment should focus on the posterior capsule.

How well did you know this?
1
Not at all
2
3
4
5
Perfectly
62
Q

A patient complains of pain in the ear, what structure does not refer to the ear?
A. Sternocleidomastoid trigger point
B. Deep masseter trigger point
C. Anterior digastric trigger point
D. Temporomandibular joint

A

C.
Anterior digastric trigger point refers to the incisors of the mandible.

How well did you know this?
1
Not at all
2
3
4
5
Perfectly
63
Q

What symptoms are indicative of a temporomandibular dysfunction problem?
A. Limited range of motion or altered mechanics
B. Tinnitus and hyperacousia
C. Dizziness and spinning
D. Retro-orbital headache and sinus pain

A

A.
Other symptoms mav include pain and tenderness located at the joint, clicking, and crepitation. Although tinnitus, headache, and dizziness mav be associated with a temporomandibular joint disorder, they are not caused by the disorder.

How well did you know this?
1
Not at all
2
3
4
5
Perfectly
64
Q

What is a reasonable rehabilitation goal for active opening after arthroscopy of the TM] for an anterior disc displacement without reduction?
A. Openingto 58 mm
B. Openingto 28 mm
C. Openingto 38 mm
D. Openingto 48 mm

A

C.
Thirty-eight millimeters is a reasonable opening range for function: eating, placing food in the mouth, brushing teeth, singing and yawning.

How well did you know this?
1
Not at all
2
3
4
5
Perfectly
65
Q

What is the best evidence based intervention for a painful anterior displaced disc with reduction in the TMI?
A. Exercises that avoid painful click such as hinge axis and midline opening
B. Aggressive mobilization to reduce the clicking
C. Wide opening exercises to reduce the clicking
D. Ice and no exercise

A

A.
Add stabilization exercises within the
click free range. These patients mavneed anterior stabilization splinting if painful clicking and catching persists.

66
Q

When is a dental splint not indicated for a TMI patient?
A. Aggressive bruxers when exercises are not enough to decrease the clench/grind habit
B. Osteoarthritic joints, bite changes
C. Muscle incoordination, bite changes, cannot find a position to rest the jaw
D. Anterior disc displacement without reduction

A

D.
A locked joint will not “unlock” by using a splint. A “locked joint” is represented by choice D.

67
Q

What is the long-term prognosis for an anterior disc displacement without reduction without surgery of the TM]?
A. Osteoarthritis will develop.
B. The range of motion will be limited to a 30 mm opening.
C. Clicking and pain will be persistent.
D. With or without surgery, the joint will adapt and have functional opening with little or no pain.

A

D.
Arthroscopy has not been found to be better than physical therapy in the treatment of reduced jaw ROM and pain due to intra-articular disease.

68
Q

What is the best intervention for an acute anterior displaced disc without reduction?
A. Ultrasound and heat to calm the joint pain and no exercises
B. Aggressive PT to manipulate joint to unlock in 3 to 4 visits
C. Referralto the dentist for splint therapy
D. Exercises to limit opening

A

B.
To alleviate an acute closed lock disorder (anterior disc displacement without reduction), aggressive PT may mobilize the joint to unlock in 3 to 4 visits. The disc may not have permanent deformation changes or adhesions form, if the patient is young and the mobilization is done early. Ifunsuccessful, additional less aggressive PT would be indicated to decrease joint inflammation, or a surgical consult may be necessary.

69
Q

Distal radius fractures
A. Are always best treated surgically
B. Are always best treated in a cast
C. Can be allowed to heal with some deformity
D. Are a “solved” problem in orthopedics

A

C.
Distal radius fractures are common and controversial. The current trend is to treat them surgically, but many patients heal with some deformitv and acceptable function.

70
Q

Stiffness at the MCP joints following casting
A. Is caused by tightness of the saggital bands
B. Can always be avoided by proper casting
C. Is unavoidable
D. Requires surgical correction

A

A.
Saggital band tightness can be difficult to resolve, but nonoperative treatment remains the treatment of choice.

71
Q

A patient sustained a Colles fracture of the wrist.
Her cast was removed and she now lacks wrist extension. To increase her passive closed chain wrist extension, mobilize the
A. Distal concave radius on the convex ulna
B. Concave distal radius on the convex proximal carpals
C. Convex distal radius on the concave distal carpals
D. Concave distal ulna on the convex proximal carpals

A

B.
The concave distal radius on the convex
proximal carpals should be mobilized. The wrist joint consists of the concave distal radius, an articular disc and the relatively convex proximal row of carpal bones. All other choices include the wrong bony surfaces.

72
Q

A pediatric patient is referred to physical therapy with a diagnosis of right torticollis. The physical therapist is developing a home program. What position does the patient need to move his head to correct his posture?
A. Right cervical rotation and left lateral cervical flexion
B. Right cervical rotation and right lateral cervical flexion
C. Left cervical rotation and right lateral cervical flexion
D. Cervical flexion and scapular retraction

A

A.
Right cervical rotation and left lateral cervical flexion is the correct choice. To correct the right torticollis the patient needs to move into right rotation and left lateral flexion to stretch his right sternocleidomastiod (SCM).

73
Q

How much cervical spine rotation will your patient have after a complete cervical spine fusion at level C1-C2?
A. Complete loss of cervical rotation
B. 25loss of cervical rotation
C. 50loss of cervical rotation
D. Noloss of cervical rotation

A

C.
The patient will have 508#37; loss of cervical rotation. The C1-C2 level produces approximately 50 of the cervical rotation allowed in the entire cervical spine.

74
Q

Your patient has limited knee flexion in the preswing phase of gait. You suspect the following
A. Hamstringweakness
B. Plantar flexor spasticity
C. Plantar flexor weakness
D. Dorsiflexor weakness

A

B.
Plantar flexor spasticity. Spasticplantar flexors will produce excessive plantar flexion, and also knee extension throughout the stance phase, and prevent adequate passive knee flexion in preswing. The hamstrings are not responsible for knee flexion in preswing phase of gait. Plantar flexor weakness would cause tibial forward collapse and therefore excess knee flexion. Dorsiflexors are not responsible for knee flexion in preswing.

75
Q

A patient demonstrates excessive hip adduction in the swing phase of gait. What are the hypothesized causes for this deviation?
A. Adductor hypertonicity of the swing leg
B. Quadriceps weakness of the swing leg
C. Plantar flexor weakness of the swing leg
D. Gluteus mediushypertonicity of the swing leg

A

A.
Adductor hypertonicity of the swing leg.
The swing leg is adducting because of hypertonicity. The quadriceps does not play a key role during swing. The plantar flexors are not active in swing; weakness is not going to create a significant problem in this phase.
Plantar flexor weakness impacts terminal stance. A person with plantar flexor weakness would demonstrate excessive hip flexion in swing, not adduction. Gluteus mediushypertonicity would cause excess hip abduction, NOT adduction.

76
Q

Your patient is recovering from ACL surgery and is 8 weeks post operative. They have difficulty with flexion beyond 10 degrees. You have determined that the most appropriate way to restore extension would be to mobilize the
A. Tibia posteriorly on the femur
B. Femur anteriorly on the tibia
C. Tibia anteriorly on the femur
D. The patella distally in the trochlear groove

A

C.
Based on the convex concave rule
concept, moving the concave surface of the tibia anteriorlv onto the convex surface of the femur will increase extension. Roll and glide will occur in the same direction. Choices A and B will both increase flexion while mobilization of the patella will be more advantages in helping with flexion of the knee because of its attachment to the quadricep muscle.

77
Q

Your patient has been diagnosed as having thoracic outlet syndrome. You perform the Adson’s maneuver and it is positive. As a result you have determined that the best way to address this problem is to stretch the
A. Pectoralis minor
B. Clavopectoral fascia
C. Anterior scalenes
D. Sternocleidomastoids

A

C.
The Adson’s maneuver tests the patency of the subclavian artery as it passes from beneath the anterior scalene and middle. If that muscle is tight, then the result will be a diminished radial pulse. Choice B would be tested by the costoclavicular test, and the pectoralis minor is assessed with the hyperabduction test. The sternocleidomastiods do not affect the thoracic outlet.

78
Q

A physical therapist is providing intervention to a 6-month-old infant diagnosed with right torticollis. The therapist instructs the family in a home exercise program that stretches the patient’s neck into
A. Left side bend and left rotation
B. Left side bend and right rotation
C. Right side bend and right rotation
D. Right side bend and left rotation

A

B.
Right torticollis involves the right sternocleidomastoid and results in cervical right side bend and left rotation. In order to stretch the muscle, the cervical spine must be placed in left side bend and right rotation.

79
Q

You are performing an examination on a 2-year-old patient diagnosed with leukemia who has been hospitalized for 1 month and is currently undergoing chemotherapy. Upon observation, you notice that the patient has difficulty with transitioning from low to high kneel. You would suspect which primary muscle is weak?
A. Biceps Femoris
B. Gastrocnemius
C. Gluteus maximus
D. Iliopsoas

A

C.
To transition from low to high kneel requires active hip extension; therefore, the gluteus maximus needs to be activated.

80
Q

The physical therapist is treating a patient with a history of coronary artery disease. During the treatment, the patient complains of recurring angina that increases when performing activities in standing. The MOST appropriate course of action by the PT is to
A. Stop treatment and contact the physician
B. Stop treatment until symptoms subside
C. Assist patient in taking medication for chest pain
D. Perform treatment in a sitting position

A

A.
Angina is recurring chest pain and is an indication of coronary artery disease. An onset of angina during treatment should be considered an emergency because of the possibility of a heart attack and will need to be addressed by the physician.

81
Q

A patient who sustained a severe heart attack was categorized at a metabolic equivalent table
(MET) level of 2 to 3. The patient has completed the goal of doing homemaking activities, such as washing dishes and ironing. The physical therapist should progress intervention to include the occupational task of
A. Driving an automobile
B. Performingupper and lower extremity dressing
C. Gardening in the yard
D. Preparing 1 to 2 meals per day

A

D.
MET refers to the amount of energy consumed at rest that is equal to the approximately equivalent to 3.5 milliliters of oxygen per kilogram of body weight per minute.
Various tasks or activities require a certain amount of energy to perform. After performing homemaking activities, such as washing dishes and ironing, the next progression in homemaking tasks from the choices listed would be preparing a meal (MET level 3 to 4).
Driving and dressing qualify as a MET level of 2 to 3; gardening has an MET level of 4 to 5.

82
Q

A patient with emphysema complains of shortness of breath and generalized weakness in the upper extremities when performing daily chores. The physical therapist should encourage
A. Pursed-lip breathing when working
B. Gravity assisted exercises before performing chores
C. Use of oxygen with daily activities
D. Avoidance of activities that consume a lot of energy

A

A.
Pursed-lip breathing technique is helpful when shortness of breath occurs.
Technique: the person inhales deeply through the nose, purses the lips as though whistling, and very slowly exhales through the lips.

83
Q

Persuading a sedentary patient to become more active, the therapist explains the benefits of exercise. Which of the following is an inappropriate list of benefits?
A. Increased efficiency of the myocardium to obtain oxygen, decreased high-density lipoprotein (HDL) cholesterol, and decreased cholesterol
B. Decreased low-density lipoprotein (LDL)
cholesterol, decreased triglycerides, and decreased resting blood pressure
C. Increased efficiency of the myocardium to obtain oxygen, decreased cholesterol, and decreased LDL
D. Decreased resting blood pressure, decreased LDL, and increased HDL

A

A.
Exercise has many benefits. Decreased
HDL in choice A makes this an inappropriate list of the benefits of exercise. HDL is considered
“good” cholesterol. Exercise decreases LDL and increases HDL in the bloodstream.

84
Q

The therapist is working in an outpatient cardiac rehabilitation facilitv. A 50-vear-old healthy man inquires about the correct exercise parameters for increasing aerobic efficiency.
Which of the following is the most correct information to convey to this individual?
A. Exercise at 80 to 90 of maximal volume of oxygen utilization (VO<font> 2</font>)
B. Exercise with heart rate between 111 and 153 beats/minute
C. Exercise at approximately 170 beats/minute
D. Exercise at level 17 to 18 on the Borg rating of perceived exertion (RPE) scale

A

B
Choice B has the patient exercising at
65to 90 of his age-adjusted maximal heart rate. Choice C is the patient’s age-adjusted maximal heart rate. Choice A is much too high a parameter. Exercise in the 65 to 90 of maximal VO<font>2</font> is much more appropriate. Patients should exercise at level 12 to 15 on the RPE scale.

85
Q

Whatlobe of the lungs is the therapist attempting to drain if the patient is in the following position? Resting on the left side, rolled one quarterturn back, supported with pillows, and the foot of the bed raised 12 to 16 inches.
A. Rightmiddle lobe-lingular segment
B. Leftupperlobe-lingular segment
C. Rightupperlobe-posterior segment
D. Leftupperlobe-posterior segment

A

A.
Choice A is the correct postural
drainage. Choice B is drained by resting on the right, one quarter turn to the back and foot of the bed elevated 12 to 16 inches. Choices C and D are drained with patient in long sitting position or leaning forward over the pillow in sitting position.

86
Q

The therapist works in a cardiac rehabilitation setting, Which of the following types of exercises are most likely to be harmful to a 64-year-old man with a history of myocardial infarction?
A. Concentric
B. Eccentric
C. Aerobic
D. Isometric

A

D.
Performing isometric exercises places too much load on the left ventricle of the heart for many cardiac patients.

87
Q

The patient is referred by a physician to begin outpatient cardiac rehabilitation. Which of the following is not a contraindication to enter an outpatient program?
A. Resting systolic blood pressure of 210 mm Hg
B. Third-degree atrioventricular block
C. Resting ST displacementless than 1 mm
D. Acute fever

A

C.
ST-segment displacement greater than
3 mm is a contraindication. Resting systolic pressure above 200 mm Hg is a contraindication.

88
Q

In order to determine if an exercise session should be terminated, the patient is asked to assess the level of exertion using the Borg Rating of Perceived Exertion Scale (RPE). The patient rates the level of exertion as 9 on the 6 to 19 scale. A rating of 9 corresponds to which of the following?
A. Very, very light
B. Very light
C. Somewhathard
D. Hard

A

B.
A rating of 9 corresponds with “very light.” A rating of 7 is “very, very light.” A rating of 13 is “somewhat hard.” A rating of 15 is
“hard.” A rating of 17 is “very hard.” A rating of 19 is “very, very hard.”

89
Q

A physician orders stage Il cardiac rehabilitation for a patient. The orders are to exercise the patient below 7 metabolic equivalents (METs). Which of the following is a contraindicated activity?
A. Riding a stationary bike at approximately 5.5 mph
B. Descending a flight of stairs independently
C. Ironing
D. Ambulate independently at 5 to 6 mph

A

D.
Riding a stationary bike at 5.5 mph is approximately 3.5 METs. Descending a flight of stairs is approximately 4 to 5 METs. Ironing is approximately 3.5 METs. Ambulating 5 to 6 mphis approximately 8.6 METs.

90
Q

A therapist is treating a patient with cystic fibrosis who has just walked 75 feet before experiencing significant breathing difficulties.
In an effort to assist the patient in regaining her normal breathing rate, the therapist gives a set of instructions. Which of the following set of instructions is appropriate?
A. “Take a slow deep breath through pursed lips, and exhale slowly through your nose onlv.”
B.”Take small breaths through vour nose only, and exhale quickly through pursed lips.”
C.”Breath in through your nose, and exhale slowly through pursed lips.”
D. “Breath in through pursed lips, and breath out slowly through pursed lips.”

A

C.
Patients with chronic obstructive
airway disease are often given this set of instructions, which is known as the method of pursed-lip breathing. This method helps a patient regain control of his or her breathing rate and increase tidal volume and amount of oxygen absorbed.

91
Q

A physical therapist is treating a 65-year-old man with COPD. The patient questions the benefits of the flow incentive spirometer left in the room by the respiratory therapist a few minutes ago. Which of the following is an appropriate response to the patient’s question?
A. “It gives visual feedback on lung performance.”
B. “You should use this for the rest of your life.”
C. “You need to ask the respiratory therapist this question.”
D. “It really doesn’t do anything useful.”

A

A.
The incentive spirometer provides visual feedback of maximal inspirator efforts.
The physical therapist is qualified to answer the patient’s question. Incentive spirometry should only be used in acute episodes of COPD. There is a risk of air trapping with long-term use.

92
Q

Which of the following is not an indication for pulmonary suctioning?
A. Unproductive coughs
B. Breath sounds of wet rales
C. Respiratory distress
D. Hyperoxygenation

A

D.
Suctioning also can be performed in patients with significant hypoxemia. All other choices are valid indications for suctioning.
Patients should be supplied with 100&# 37; oxygen before suctioning to prevent hypoxemia.

93
Q

Which of the following exercises does not increase strength of the muscles of forceful inspiration?
A. Active cervical flexion exercises
B. Active glenohumeral extension exercises
C. Shoulder shrugs
D. Crunches

A

D.
Choice A increases strength of the scalenes and sternocleidomastoid. Choice B strengthens the latissimusdorsi. Choice C increases the strength of the upper trapezius.
All of these are accessory inspiratory muscles.
Choice D strengthens the abdominals, which are muscles of forceful expiration.

94
Q

A 63-year-old man presents to an outpatient physical therapy clinic with a diagnosis of sciatica. The MRI report is negative for lumbar disc involvement. During the examination the physical therapist cannot reproduce the symptoms of radiculopathy with any test.
Lower extremity strength is equal bilaterally and is not weak in any particular pattern. The patient informs the therapist that the pain is bilateral, located in the gastrocnemius area, and increases with prolonged ambulation. The pain stops soon afterresting in a seated position.
What is the most likely source of this patient’s
pain?
A. Impingement of the L5 dorsal root
B. Multiple sclerosis
C. Compartment syndrome
D. Intermittent claudication

A

D.
Intermittent claudication is a sign of chronic arterial disease. Choice A is incorrect because it produces unilateral signs and symptoms. Choice B is incorrect because, although the signs and symptoms may be present in a patient with multiple sclerosis, this scenario paints a more accurate picture of a patient who has intermittent claudication. A compartment syndrome usually involves the anterior tibialis. In addition, patients with compartment syndrome require a longer rest time than this question implies before pain subsides.

95
Q

An 81-year-old woman with right-side hemiparesis due to stroke is being treated by a physical therapist through home health services.
The therapist is attempting to increase the functional reach of the right upper extremity.
The patient currently has 120 degrees of active flexion. The therapist decides to use trunk mobility/stability facilitation techniques to help achieve the patient’s functional goals. Which of the following skills need to be mastered by the patient to attain the ability to reach 2 feet in front of her wheelchair and 2 feet to the right of midline at 125 degrees of shoulder flexion with the right upper extremity?
A. Weight shifting to the left buttock and right-side trunk elongation
B. Weight shifting to the left buttock and left-side trunk elongation
C. Weight shifting to the right buttock and right-side trunk elongation
D. Weight shifting to the right buttock and left-side trunk elongation

A

C.
To reach as described in the question, the patient must shift weight to the right buttock and elongate the right side of the trunk.
With the same circumstances given in the question, but to the left side, the patient would shift weight to the left buttock and elongate the left side of the trunk.

96
Q

The physical therapist in beginning examination of a patient in an outpatient cardiac rehabilitation facility. A chart review shows that this patient has active atrial fibrillation with a controlled ventricular rate. What is the most appropriate intervention for this patient?
A. Low intensity aerobic exercise
B. High intensity aerobic exercise
C. High intensity lower extremity exercise onlv
D. Low intensity lower extremity exercise onlv

A

A.
Atrial fibrillation is a relative contraindication for therapy. Exercise should start at a lower intensity and be progressed slowlv if the ventricular rate remains controlled.
There is no contraindication against upper extremity exercise.

97
Q

Which of the following is incorrect advice to give to a patient with a diagnosis of congestive heart failure who complains of shortness of breath and “smothering” while attempting to sleep?
A. Sleep with the head on 2 to 3 pillows.
B. Sleep without any pillows.
C. Sleep in a recliner during exacerbations.
D. During exacerbations, come to a standing position for short-term relief.

A

B.
In complete supine, patients with this diagnosis will have excess fluid move from the lower body to the chest cavity. This causes a decrease in heart and lung function and efficiency.

98
Q

A physical therapist is working with a patient who has a historv of coronar arterv disease and a reported history of chest pain. Which of the following from the medical interview is NOT a reason to refer a patient back to their physician for further follow-up?
A. Reported anginal pain is not relieved by rest.
B. Chest pain is reported lasting 20 minutes or longer.
C. There is a reported change in the pattern of angina.
D. Nitroglycerine tablets are reported to relieve the angina.

A

D.
According to Goodman and Snyder
(Differential Diagnosis in Physical Therapy, 3rd edition), Choices A through C are red flags and indicate the need for immediate referral and follow-up for patients with a histor of cardiac dysfunction. There is a concern that the patient is not medically stable or there is an expansion of the infarction. Exercise at present is contraindicated. Chest pain that is managed effectively with nitroglycerin is not an immediate cause for concern.

99
Q

All of the following cardiopulmonary function variables will increase in children in response to training except
A. Heart volume
B. Respiratory rate
C. Stroke volume
D. Tidal volume

A

B.
Training effects usually include
increases in myocardial mass, stroke volume, ventilation, and respiratory muscular endurance.

100
Q

You are performing an examination and providing intervention to a patient diagnosed with hemophilia. You have confirmed that hemostasis has been achieved after an acute bleed in the right knee. During the objective examination, you determine that the knee has 100 degrees of knee flexion and there is a 5 degrees knee flexion contracture. All are appropriate therapeutic interventions except
A. Passive ROM of the right knee
B. Instruct patient and family in a home exercise program consisting of AROM and strengthening
C. Active resisted exercises
D. Isometrics of the quadriceps muscle

A

A.
Once hemostasis has been
achieved after an acute hemorrhage, the physical therapy program should include instruction of a home exercise program consisting of active ROM and strengthening exercises, isometrics, knee extension exercises in supine and sitting, and active resistive exercises mav be initiated if the knee has at least 90 degrees of flexion andless than 15 degrees flexion contracture. Passive ROM is contraindicated.

101
Q

A patient with reflex sympathetic
dystrophy syndrome (RSDS), or complex regional pain syndrome (CRPS), has severe edema and increased pain while performing simple grooming activities such as face washing.
The initial care plan should focus on
A. Pain management
B. Using the uninvolved extremity
C. Gentle mobilization
D. Light activities, such as brushing teeth

A

A.
The PT should work to control pain before using the involved extremity for functional activities, such as grooming and basic ADLs. Although gentle mobilization may be used, pain management techniques should be the immediate focus. Pain management spans a wide array of techniques, including relaxation, massage, biofeedback, and positioning.

102
Q

A 45-year-old patient is in the first stage of RSDS or CRPS with the classic signs of hand edema, fear of ROM, and pain in the shoulder and hand of the involved extremity.
What is the FIRST priority of treatment?
A. Reduction of edema
B. Aggressive PROM
C. Instruct in self-ROM
D. Perform grip strength test

A

A.
The first priority is to reduce
edema because other symptoms may be somewhat relieved and motion can be enhanced with a reduction in swelling.

103
Q

A patient with a spinal cord injurv at
the level of C8 would like to be independent in mobility. Based on the expected functional outcomes, the physical therapist would recommend the following piece(s) of adapted equipment
A. Manual wheelchair
B. A motorized wheelchair
C. A manual wheelchair and sliding board
D. A walker and wheelchair

A

C.
If there are no other
complications, a person with a spinal cord injurv at the level of C8 would most likely be independent in mobility with the use of either a manual or motorized wheelchair and a sliding board for transfers.

104
Q

A PT working in early intervention is
helping a parent to get the baby to hold and drink from a bottle. Based on typical development, the therapist should beginto introduce this skill between
A. 12 to 14 months
B. 10 to 12 months
C. 8 to 10 months
D. 6 to 8 months

A

D. The skill of holding and drinking from a bottle typically emerges around
6 months of age.

105
Q

After 3 months of intervention, the
physical therapist notices that the child is beginning to integrate the reflex that turns the head toward the child’s extended arm while in prone. This reflex is
A. Asymmetric tonic neck reflex
B. Symmetric tonic neck reflex
C. Moro reflex
D. Tonic labyrinthine reflex

A

A.
The asvmmetric tonic neck
reflex (ATNR) is present in utero through 6 to 8
months while the child is awake and up to 42 months while the child is sleeping. Because of the ATNR reflex the child’s head turns toward the extended arm and leg and the opposite arm andleg bend. This reflex may help in the birth process, assist in the development of visual motor integration, and protect the airway while the child is in the prone position.

106
Q

A physical therapist arrives at a child’s
home to begin a treatment session. When the therapist enters the home, the child is running through the house screeching. The PT determines that the first step in the intervention session will be
A. Sitting still on a therapy ball
B. Placing the child in a high chair
C. Rhythmic linear swinging
D. Rapidly brushing extremities

A

C.
Rhythmic swinging calms the
child while maintaining a level of alertness. This is an effective use of sensory preparation for activity. Combine this with rhythmic vocalization and deep pressure to help calm the child before engaging in an activity. The therapist must watch the child for response to the sensory preparation and gear input accordingly.

107
Q

A child has poor ability to maintain trunk and neck extension. The PT uses which of the following as the best technique to facilitate increased strength and control
A. Have child prone on a therapy ball and plav with tovs
B. Have child supine on a platform swing while playing with toys
C. Have child in side lying on a mat shield playing with toys
D. Have child sit on physiob all while playing with toys

A

A.
The prone position is the best
for facilitating neck and trunk extension whether on a ball, bolster, or wedge.

108
Q

A PT is working with a child who has
cerebral palsy. The child has limited range of motion (ROM) in bilateral upper extremities and is unable to reach out for obiects. The PT provides intervention that focuses on allowing the child to participate in play activities. The best position to place the child in is
A. Side-lying
B. Prone
C. Supine
D. Sitting

A

D.
The most common and effective
position to place a child in is sitting, with attention given to head and neck control, visual regard, and visual tracking. Although the child can be placed in the supine and side-Iving positions, sitting is the most commonly used position.

109
Q

A child diagnosed with cerebral palsy
has severe spasticity in the bilateral upper extremities. The occupational therapv referral states “fabricate splints to prevent hand deformities.” The theoretical approach for splinting should emphasize placement of the hands in the
A. Intrinsic minus position
B. Anticlaw position
C. Resting hand position
D. Reflex inhibiting position

A

D.
The NDT approach advocates
the use of the reflex-inhibiting patterns to inhibit spasticity. Finger and thumb abduction are key to controlling spasticity by facilitating extensor muscle tone and inhibiting flexor tone.

110
Q

A spinal cord injury patient who can
breathe on his own, uses a sip and puff switch to operate his power wheelchair and environmental control unit, and a mouth stick for writing, table games, and the computer would MOST likely have a spinal cord injury at this level.
A. C3
B. C4
C. C5
D. C6

A

B.
A client with a C4 injury would
have innervation of the muscles controlling the head, neck, and diaphragm and would be the most appropriate choice for these activities.

111
Q

The highest level of spinal cord injury at which vou would expect a client to become independent in all self-care and driving with equipment would be
A. C7
B. C8
C. C5
D. C6

A

D.
A client with a C6 injury would have control of the head, neck, diaphragm. deltoids, biceps, and wrist extensors, giving the client enough function to complete the activities.

112
Q

You are treating a client that recently had a spinal cord injury at C5. The BEST piece of equipment to help the client with feeding, hygiene, grooming, writing, and driving a power wheelchair would be
A. Deltoid assist
B. Weighted pulley
C. Mobile arm supports
D. Suspension sling

A

C.
Mobile arm supports would be
the best choice because they can be mounted to the wheelchair and will allow for the most ease of movement in a safe manner.

113
Q

The MOST beneficial piece of adaptive equipment to aid a client with C6 tetraplegiato work toward independence with lower extremity dressing is
A. A trigger reacher to get the pants over the feet
B. Loops to manipulate the lower extremities
C. A universal cuff to assist with grip and closures
D. A standard dressing stick

A

B.
The loops would be the best.
The client would be unable to use a trigger reacher and the standard dressing stick because the client does not have a functional grasp. The universal cuff would be of no benefit to grip and don pants and could only be used for closure with an additional device (button hook/zipper pull).

114
Q

As a home health therapist, you are treating a 55-year-old who has a very supportive spouse and a caregiver during the day who helps with self-care and other tasks needed in the home. The patient enjoys their children and grandchildren who live in the immediate area. The patient is currently in stage 3 of amyotrophic lateral sclerosis, with severe weakness of the ankles, wrists, and hands. The patient minimally ambulates and fatigues easily. An appropriate intervention would be
A. Light strengthening program
B. Help prioritize activities and provide work simplification
C. Learning how to cook three-course meals
D. Worksite assessment

A

B.
Amyotrophiclateral sclerosis is
a degenerative disease without a cure that results in death. Stage 3 is characterized by moderate dependence in self-care ADLs and IADLs along with severe weakness of the arms andlegs. At this stage of the disease, conserving energy and quality of life are paramount.

115
Q

A 3-year-old has spina bifida and needs
mobility augmentation to be able to move outdoors, in hallways, and in corridors. A mobility device that could be recommended would be a(n)
A. Hand-propelled tricycle model
B. Supine scooter
C. Aeroplane mobility device
D. Crocodile posterior walker

A

A.
Hand-propelled tricycle models
are available for children who do not have the ability to pedal with their legs. These can provide mobility outdoors, in hallways, and in corridors.

116
Q

You are visiting the home of a client recently diagnosed with Alzheimer’s. The environment is terribly cluttered and seems to add to the patient’s current level of confusion.
You decide to
A. Clean up the place by yourself, throw away a lot, and put everything else away properly to help organize the environment for the client
B. Leave it alone and just recognize that this is how the person will be living
C. Engage family members in helping the client to sort through some of the cluttered items and make some choices about what to and what not to keep
D. Encourage the family to hire a housekeeper, who will make sure that the environment is clean and tidy all the time

A

C.
Alzheimer’s patients often have
difficulty with change. The family members can contribute information about what items are valuable and should be retained and can also assist the client in gradually changing in the environment to reduce stress and allow time to adjust.

117
Q

Which of the following is the most important to assess first during an examination of a patient with a recent stroke?
A. Sensor status
B. Motor control
C. Mental status
D. Ambulation potential

A

C.
Mental status is the first item to
assess. A therapist must first determine whether the patient is able to provide a reliable subiective history. It is also important to know whether the patient can follow a 1- or 2-step command before beginning a formal evaluation.
The other choices should be assessed later in the evaluation.

118
Q

A therapist receives an order to examine a 72-vear-old woman who has suffered a recent stroke. The therapist needs to focus on pregait activities. Which of the proprioceptive neuromuscular facilitation (PNF) diagonals best encourages normal gait?
A. D1
B. D2
C. PNF is contraindicated
D. Pelvic PNF patterns only

A

A.
The therapist would use a PNF
D1 diagonal to encourage the combined movements of hip flexion, adduction, and knee flexion. The diagonal also encourages the combined movements of hip adduction and extension. This is the combination of muscle activity most needed for gait.

119
Q

The following is a long-term goal for a patient with a spinal cord injury: independence in performing a manual cough without applying pressure to the abdomen. This goal is the most challenging and obtainable for a patient with a complete lesion at which of the following spinal cord levels?
A. C5
B. C7
C. T2
D. T10

A

B.
A patient with a spinal cord
injury at the C5 level would apply pressure to the abdomen to perform a cough. A patient with an injurv at the T2 level and T10 level should be able to perform a cough independently, but this goal would be most challenging and obtainable for a patient with an injury at the level of C7.

120
Q

To treat effectivelv most patients with
Parkinson’s disease, the therapist should emphasize which proprioceptive
neuromuscular facilitation (PNF) pattern for the upper extremities?
A. D2 extension
B. D2 flexion
C. D1 extension
D. D1 flexion

A

B.
D2 flexion patterns support
upper trunkextension, which is important for patients with Parkinson’s disease who tend to develop excessive kyphosis.

121
Q

Which of the following is the most
energy efficient and allows a T1 complete paraplegic the most functional mobility during locomotion?
A. Manual wheelchair
B. Electricwheelchair
C. Bilateral knee-ankle orthoses and crutches
D. Bilateral ankle-foot orthoses and crutches

A

A.
An electric wheelchair
definitely uses less energy but does not require the physical effort needed by this patient to maintain functional mobility. Ambulation with a knee-ankle-foot orthosis is probably possible but requires much more energy than locomotion with a manual wheelchair. Ankle-foot orthoses alone do not provide enough support for the patient to attempt ambulation.

122
Q

To facilitate development of a
functional tenodesis grip in a patient with a spinal cord injury, the treatment plan should include
A. Stretching of the finger flexors and finger extensors
B. Stretching of the finger flexors
C. Allowing the finger flexors and finger extensors to shorten
D. Allowing the finger flexors to shorten

A

D.
To assist a patient in developing
a tenodesis grip, the therapist should allow the patient’s finger flexors to tighten. This grip functions with active extension of the wrist, which allows flexion of the fingers because of shortened flexor tendons.

123
Q

A therapist is treating a patient with an injury at the T8 level and compromised function of the diaphragm. If no abdominal binder is available, what is the most likely position of comfort to allow him to breathe most efficiently?
A. Sitting position
B. Semi-Fowler’s position
C. Upright standing position using a tilt table
D. Supine

A

D.
Choice D is the correct answer
because in the supine position the abdominal contents are located more superiorly than in the other positions. This places the diaphragm in a more elevated resting position, which allows greater excursion of the diaphragm.
Semi-Fowler’s position resembles a reclining position, with the knees bent and the upper trunk slightlv elevated. Semi-Fowler’s position, without an abdominal binder, allows gravity to pull the abdominal contents downward, which does not put the diaphragm in an optimal resting position. Semi-Fowler’s position is, however, the position of choice for patients with uncompromised innervation of the diaphragm who have chronic respiratory difficulty. The standing and sitting positions present the same problem, but to a greater extent, as semi-Fowler’s position.

124
Q

A therapist is assisting a patient with an injury at the C5 level in performing an effective cough. The patient has experienced significant neurologic damage and is unable to perform an independent, effective cough. Ifthe patient is in a supine position, which of the following methods is most likely to produce an effective cough?
A. The therapist places the heel of one hand just above the xiphoid process, instructs the patient to take a deep breath while pressing down moderately on the sternum, and instructs the patient to cough.
B. The therapist places the heel of one hand, reinforced with the other hand, just above the xiphoid process, instructs the patient to take a deep breath.
instructs the patient to hold the breath, and presses moderately as the patient coughs.
C. The therapist places the heel of one hand on the area just above the umbilicus, instructs the patient to take a deep breath, applies moderate pressure, and releases pressure just before the patient attempts to cough.
D. The therapist places the heel of one hand just above the umbilicus, instructs the patient to take a deep breath, and applies moderate pressure as the patient is instructed to cough.

A

D.
The pressure applied by the
therapist should be applied as the patient coughs to assist in a forceful exhalation. Placing the heel of one hand approximately one inch above the umbilicus applies pressure immediately inferior to the diaphragm.

125
Q

A physical therapist is ordered to examine a 65-vear-old woman who has suffered a recent stroke. The occupational therapist informs the physical therapist that the patient has apraxia. She cannot brush herteeth on command. However, she can point out the toothbrush and verbalize the purpose of the brush. From this information, what sort of apraxia does this patient have? How should the physical therapist approach treatment?
A. Ideomotor apraxia; The physical therapist should speak in short, concise sentences.
B. Ideational apraxia; The physical therapist should always give the patient
3-step commands.
C. Ideomotor apraxia: The physical therapist should always give the patient
3-step commands.
D. Ideational apraxia: The physical therapist should speak in short, concise sentences.

A

A.
Patients with ideomotor
apraxia often can identifv obiects but cannot use them correctly on command. Such patients often can perform the activity spontaneously. Patients with ideational apraxia often cannot identify obiects or use them. Both situations call for short one-step commands.

126
Q

A 60-year-old woman who has suffered a recent stroke has right-side homonymous hemianopsia. Which of the following statements is true about placement of eating utensils in early rehabilitation?
A. The utensils should be placed on the left side of the plate.
B. The utensils should be placed on the right side of the plate.
C. The utensils should be placed on both sides of the plate.
D. The plate and utensils should be placed slightly to the right.

A

A.
As perception improves, objects
should be moved into the area of the deficit (the right side in this case, but initially they should be placed in plain view of the patient (the left side in this case).

127
Q

A 25-year-old man suffered C4
quadriplegia in a motor vehicle accident. The injury is acute, and the patient is beginning to work on increasing upright tolerance in the sitting position with an abdominal binder. He is looking to the therapist for encouragement. The therapist is attempting to convey realistic long-term goals for self-care ability and overall mobility. Of the listed goals, what can this patient reasonably expect at his highest level of function in the future?
A. Transfer from wheelchair to bed independently with a sliding board
B. Use of a powerwheelchair
C. Independent feeding without an assistive device
D. Donning a shirt independently and pants with minimal assistance

A

B. A person with C4 quadriplegia can be reasonably expected to use a power wheelchair for locomotion with mouth, chin. breath, or sip-and-puff controls. A person with C5 quadriplegia may be reasonably expected to be able to transfer independently from wheelchair to bed with a sliding board. A person with C4 quadriplegia maybe able to feed independently but will need some type of assistive device. A person with C5 quadriplegia may be able to don a shirt with assistance.
Sources vary significantly on this subject.

128
Q

A 17-year-old boy presents to therapy afterbeing involved in a motor vehicle accident resulting in C7 quadriplegia. The therapist is settinglong-term goals for the patient. Which of the following goals represents the most reasonable and highest level of function that the patient should achieve?
A. Use of a wheelchair with power hand controls on eventerrain
B. Negotiation of uneven terrain with a manual wheelchair
C. Ambulation for short distances on level surfaces with knee-ankle-foot orthoses
D. Use of a power wheelchair with head or chin controls on even surfaces

A

B.
A person with C7 quadriplegia should be able to use a wheelchair without power controls. The goals set in Choices A and D do not represent the maximal functional potential for this patient. The goal in Choice C is set too high for this patient.

129
Q

A therapist is treating a patient with a spinal cord injury. The therapist is discharging the patient after completion of all physical therapy goals. One of the completed long-term goals involved the ability to dress and bathe independently with assistive devices. This would be a most challenging but obtainable goal for which of the following?
A. C5 quadriplegia
B. C7 quadriplegia
C. T1 paraplegia
D. C4 quadriplegia

A

B.
This goal should be most
challenging and obtainable for a patient with C7
quadriplegia. Aperson with C4 or C5 quadriplegia probably needs assistance from another person to dress and bathe. A person with C7 quadriplegia would find this goal more challenging than a person with T1 paraplegia.

130
Q

A physical therapist is examining a 5-day-old infant with cerebral palsy. The infant has an abnormal amount of extensor tone.
Which of the following is incorrect positioning advice for the family and nursing staft?
A. Keep the infant in a supine position.
B. Keepthe infant in a prone position.
C. Keepthe infant in a right side-lying position.
D. Keepthe infant in left side-lying position.

A

A.
Prone and side-lying positions would encourage flexion of the extremities with this patient. In this population, prone positioning allows more efficient cardiovascular function. The right or left side of side-lying does not make an difference in this situation.

131
Q

A physical therapist is treating a 76-year-old woman with left lower extremity hypotonia secondary to a recent stroke. Which of the following is an incorrect method to normalize tone?
A. Rapid irregular movements
B. Approximation
C. Prolonged stretch
D. Tactile cues

A

C.
A prolonged stretch assists in decreasing tone.

132
Q

A therapist is attempting to open the spastic and flexed hand of a patient who has suffered a recent stroke. Which of the following does not inhibit hand opening?
A. Avoid touching the interossei.
B. Apply direct pressure to the thenar eminence.
C. Hyperextend the metacarpophalangeal joint.
D. Apply direct pressure to the hypothenar eminence.

A

A.
Avoiding the interossei helps to
inhibit tone. Direct pressure to any hand musculature may increase tone. Hyperextension of the MCP joints also may cause an increase in tone.

133
Q

Which of the following is inappropriate for a physical therapist to include in the treatment plan of an infant with a gestational age of 27 weeks and Down’s syndrome?
A. Bottle feeding
B. Encourage side-lying position
C. Tactile stimulation with the entire hand rather than the fingertips of the examiner
D. Prone positioning

A

A.
Bottle or breastfeeding is rarely
performed successfully before 34 weeks of gestational age. Side-lying position allows the infant to move the hands toward the mouth.
The prone position encourages flexion. Full contact with the hand is more comforting to the infant.

134
Q

Which of the following sources of stimulation is least effective in obtaining functional goals when treating an infant with decreased muscular tone?
A. Vestibular
B. Weight bearing
C. Cutaneous
D. Vibratory

A

D.
Although vibration often elicits
a muscle contraction, a therapist should first choose stimulithat are more likelv to occur naturallv.

135
Q

Which of the following is the most important goal in treating pediatric patients with postural reaction deficits?
A. Age-appropriate responses
B. Automatic responses
C. Conscious responses
D. Lower extremity control before upper extremity control

A

B.
Postural reactions are automatic unconscious reactions to changes in center of mass. Choice A is an appropriate goal but not alwavs the most important.

136
Q

A physical therapist is treating an 81-
year-old man with Parkinson’s disease. The patient has been ambulating with a cane. He was referred to physical therapy because of a fall at home. The family reports a decrease in gait ability during the past several months. The therapist decides to begin gait training with a rolling walker. Which of the following is incorrect for the treatment of this patient?
A. Strengthening of the hip flexors, and stretching of the gluteals
B. Slow, rhythmical rocking techniques
C. Biofeedback during ambulation
D. Prolonged passive stretching of the gastrocnemius muscle group bilaterally

A

A.
Patients with Parkinson’s
disease usually ambulate with the trunk in flexion. Increased trunk flexion causes a festinating gait to be more pronounced.
Therapy should strengthen extensor muscles while stretching the flexors. Slow rocking has been shown to decrease tone, and biofeedback can improve a gait with shorter step and stride length by placement of markers on the floor for the feet.

137
Q

A patient who has suffered a recent stroke is being treated by a physical therapist.
The patient exhibits increased extensor tone in the supine position along with an exaggerated symmetric tonic labyrinthine reflex (STLR).
What position should be avoided if the therapist is attempting to initiate flexion movements of the lower extremity?
A. Prone position
B. Right side-lying position
C. Supine position
D. Left side-lying position

A

C.
When an exaggerated
symmetric tonic labyrinthine reflex is present, supine positioning increases extensor tone and prone positioning increases flexor tone. Side-Iving also provides an opportunity for the physical therapist to stimulate flexion. Right of left side-lying makes no difference in this case.

138
Q

A patient presents to outpatient physical therapy with tarsal tunnel syndrome.
What nerve is involved? Where should the therapist concentrate treatment?
A. Superficial peroneal nerve, inferior to the medial malleolus
B. Posterior tibial nerve, inferior to the medial malleolus
C. Superficial peroneal nerve, inferior to the lateral malleolus
D. Posterior tibial nerve, inferior to the lateral malleolus

A

B.
Tarsal tunnel syndrome is
caused by compression of the posterior tibial nerve as it travels through the tarsal tunnel. The tarsal tunnel is formed by the medial malleolus, medial collateral ligament, talus, and calcaneus.

139
Q

Which of the following positions should be avoided in the right upper extremity with a patient who has a diagnosis of right hemiplegia secondary to a stroke?
A. Prolonged shoulder adduction, internal rotation, and elbow flexion
B. Prolonged shoulder abduction, internal rotation. and elbow flexion
C. Prolonged finger and thumb flexion
D. Prolonged wrist flexion and finger adduction

A

B.
Flexed postures should be
avoided with this patient population. Positions of shoulder adduction, internal rotation, and wrist flexion are contraindicated. As well as wrist, finger, thumb flexion, and finger thumb adduction.

140
Q

A 35-year-old male patient has a diagnosis of right C5-C6 cervical nerve root compression. He is being seen in physical therapy for gentle manual cervical traction.
What position is ideal for traction with this patient?
A. Upper cervical flexion andlower cervical extension
B. Cervical lateral flexion to right
C. Cervical extension
D. Cervical flexion

A

D.
Cervical flexion. Cervical flexion
opens up the cervical intervertebral joint spaces.
Any extension or lateral flexion towards the impingement will result in nerve root compression.

141
Q

During physical therapy sessions, the physical therapist protects tenodesis of the patient with diagnosis of C6 tetraplegia. What position must be maintained during upper extremity weight bearing and why?
A. Maintain finger flexion with wrist extension to protect extrinsic wrist extensors.
B. Maintain wrist flexion to protect intrinsic finger extensors.
C. Maintain finger flexion to protect extrinsic finger flexors.
D. Maintain wrist extension to protect intrinsic finger flexors.

A

C.
Maintain finger flexion to
protect extrinsic finger flexors. Tenodesis is passive insufficiency of the extrinsic finger flexors. After SCI lesion at C6, people can use preserved wrist extension combined with passive finger flexion to grip objects. Preserving extrinsic finger flexortightness is essential to maintaining passive insufficiency.

142
Q

A patient on the acute rehabilitation unit was diagnosed with traumatic closed head injury. He is functioning at a Rancho Cognitive Level of Function IV. He has no significant muscle weakness or gait impairments and walks without assistance. The team needs physical therapy’s input on recommendations for safety and supervision. The physical therapist recommends
A. No supervision is needed as the patient walks safelv
B. Constant supervision is needed because of confusion and possible agitation
C. Intermittent supervision is needed to monitor his ability to follow a schedule
D. Occasional supervision is needed when faced with new situations or schedule changes

A

B.
Constant supervision is needed
due to confusion and possible agitation. Rancho Los Amigos Cognitive Level of Function IV is confused-agitated and will need constant supervision.

143
Q

Which is not a typical clinical finding of a patient with a brachial plexus injurv?
A. Decreased ROM/contractures
B. Decreased muscle strength
C. Spasticity
D. Altered sensation

A

C.
A brachial plexus injury is a
lower motor neuron injurv; and therefore, it does not cause spasticity, an upper motor neuron sign.

144
Q

All are appropriate physical therapy goals for a 6-month-old patient with a brachial plexus injury except
A. Able to creep forward 5 cycles
B. Able to demonstrate protective reactions to each side in sitting
C. Able to prop on forearms in a prone position with the head elevated 90 degrees
D. Family independent with a home exercise program consisting of ROM and positioning

A

A.
All are appropriate goals and
gross motor skills for a 6-month-old with a brachial plexus injury except creeping forward, which is more appropriate for an 8- to 9-month-old.

145
Q

What would be the most appropriate
intervention to include in the physical therapy plan of care for a 7-year-old female diagnosed with myelomeningocele at the T10-T11 level?
A. Aggressive ROM of the lower extremities
B. Gait training with a HKAFO and walker
C. Gaittraining with a KAF0 and forearm crutches
D. Gait training with a reciprocating gait orthosis

A

D. A patient with myelomeningocele at the T10-T11 level would have innervation of the abdominals but not of the lower extremities. Therefore, the patient would require a reciprocating gait orthosis in order to ambulate. In addition, aggressive ROM of the lower extremities would be contraindicated for a patient with this diagnosis because of possible osteoporosis and risk for fracture.

146
Q

An appropriate physical therapy
program for an infant with a brachial plexus injury during the second week of life would include
A. Teaching the parents how to support the arm while dressing and moving the infant to prevent further injury
B. Daily PROM exercises including full shoulder abduction and elevation
C. Daily AROM exercises including full shoulder abduction and elevation
D. Splinting the shoulder in abduction and internal rotation

A

A.
The physical therapy program
should focus on preserving function and minimizing further injury to the affected extremity. Nursing staff and parents need to be taught how to handle the patient during dressing and transfers to reduce stress and pain in the arm. Passive ROM is important and should be initiated around week 1 but no later than 3 weeks. Shoulder abduction and elevation is limited to 90 degrees for the first 3 weeks. It is no longer recommended that the arm be splinted in abduction and external rotation position because of the risk of causing shoulder dislocations.

147
Q

Which of the following exercise
programs is most appropriate for a 62-year-old, postmenopausal female patient with MS who has poor balance and decreased strength?
A. Stretching, posture, and balance program, with strengthening exercises as tolerated
B. Stretching and progressive hill training on treadmill
C. Warm water aquatic therapy
D. Progressive hill training on treadmill with plyometric training

A

A.
Activities that unduly increase
body temperature are not recommended for patients with MS. Keep in mind that spasticity is a significant complication of MS and can adversely affect gait parameters. If the patient is presenting with poor balance and decreased strength, hill training and plyometric training may be too aggressive for this particular patient.
At this age, improving balance, strength, and coordination are paramount to preventing falls and future injury.

148
Q

A therapist is instructed to provide
electrical stimulation to a patient with a venous stasis ulcer on the right lower extremity. What is the correct tpe of electrical stimulation to promote wound healing?
A. Biphasic pulsed current
B. Direct current
C. Interferential current
D. Transcutaneous electrical stimulation

A

B.
Direct current is shown to have
the greatest benefit in wound healing.
Monophasic pulsed current has also been shown to have wound healing benefits.

149
Q

Which of the following is the best and first treatment for a wound with black eschar over 90 of the wound bed?
A. Lidocaine
B. Dexamethasone
C. Silvadene
D. Elase

A

D.
The correct treatment involves
debridement of the eschar over the wound.
Elase is an enzymatic wound debridement ointment. Lidocaine is an anesthetic.
Dexamethasone is a steroid used mainly with iontophoresis. Silvadene is an antimicrobial used to prevent infection.

150
Q

A therapist is treating an acute full-thickness burn on the entire right lower extremity of a 27-year-old man. What movements need to be stressed with splinting, positioning, and exercise to avoid contractures?
A. Hip flexion, knee extension, and ankle dorsiflexion
B. Hip extension, knee flexion, and ankle plantar flexion
C. Hip extension, knee extension, and ankle dorsiflexion
D. Hip flexion, knee extension, and ankle plantar flexion

A

C.
This answer is correct because
the most common deformity after a severe burn such as this is hip flexion, hip adduction, knee flexion, and ankle plantar flexion.

151
Q

A therapist is teaching a family how to care for a family member at home. The patient is totally bed-bound. To prevent pressure ulcers most effectively, what should be the maximal amount of time between position changes?
A. 1 hour
B. 2 hours
C. 6 hours
D. 8 hours

A

B.
To prevent pressure (decubitis)
ulcers effectively, patients should be turned every 2 hours.

152
Q

Which of the following does not facilitate ambulation when the feet are burned?
A. Constant movement, avoiding standing still
B. Loosening or removing the bandages /wraps
C. Establishing a clear goal for walking (e.g., to a favorite person/place)
D. Exercise before standing upright

A

B.
Choices A, C and D are key to
early ambulation after a burn. Bandages should not be loosened unless they are painful. Loose bandages might cause edema.

153
Q

In which of the following wounds would the physical therapist consider using enzymatic debridement?
A. Black eschar over the wound
B. Cleanwound
C. A dryischemic wound
D. A drywound with active infection

A

A.
Enzymatic debridement is only
appropriate in wounds that have thick black eschar. The enzymes have no effect on infections.

154
Q

What is the most correct ambient
temperature for a room that normally has a predominant population of burn patients?
A. 65° F
B. 72° F
C. 78° F
D. 85° F

A

D.
Burn patients lose heat more
rapidly than other individuals. It is advisable to keep room temperatures at a higher than normal level.

155
Q

A 47-year-old man with end-stage renal
disease arrives at an outpatient facility. He has a physician’s order to examine and treat 3 times/week for 4 weeks secondary to lower extremity weakness. The patient also attends dialvsis 3 times a week. If the clinic is open Monday through Friday, which of the following schedules is appropriate?
A. On the davs that the patient has dialvsis, schedule the therapy session before the dialysis appointment.
B. On the davs that the patient has dialvsis, schedule the therapy session after the dialysis appointment.
C. Contact the physician and obtain a new order to decrease the frequency to 2 times/week.
D. Contact the physician and recommend home health physical therapy because outpatient therapy may be too aggressive.

A

A.
Dialysis leads to a change in
blood chemistry and volume, often causing extreme fatigue, treatments should be before dialysis. There is no need to decrease frequency or recommend home health.

156
Q

Though relapses associated with
multiple sclerosis (MS) tend to decrease during pregnancy, postpartum they increase. As physical therapists we are concerned with these patients’ __________ and to care for their new baby.
A. sleeping patterns, strength, and attention
B. vision, strength, and financing
C. balance, fatigue, and ability
D. balance, posture, and attention

A

C.
Most women experience relief
of most, if not all MS symptoms during pregnancy. However, relapse rate postpartum is considered to be 20 to 40.Itis recommended that patients with MS have a contingency plan in place to plan for changes in the ability to take care of the baby.

157
Q

Which is NOT a maternal response to mild-to-moderate exercise?
A. Increased cardiac output
B. Increased stroke volume
C. Normalto increased heart rate
D. Decreased respiratory rate

A

D.
As pregnancy advances, oxygen
and carbon dioxide have more challenge transferring from the air to cells. Though there is increased cardiac output, there is also increased demand. Pregnant women compensate for this by breathing more deeply and with increased frequency.

158
Q

You are working in home health and visit a woman who is 8 months pregnant. The doctor has just placed her on strict bed rest; she is only allowed up to go to the bathroom. In what position will you have the patient do most of her exercise?
A. Standing
B. Supine
C. Prone
D. Side-lying

A

D.
Usually assigned to allow
improved uterine blood flow, bed rest can have devastating effects on a mother’s body and mind. It is recommended to do exercise in side-lying to limit stress/pressure on the inferior vena cava possibly achieved in supine positioning prone positioning at this stage in pregnancy is impractical, standing may be too stressful on the cervix and uterus.

159
Q

Three weeks earlier, your patient
delivered via C-section a healthy baby girl who weighed in at 8 pounds, 6 ounces. This was her first pregnancy/delivery. Besides the incisional pain, she is having trouble &quot:making it to the bathroom" and is very nervous of reaching over to pick up the baby. Which combination of activities is appropriate to do/complete on her first visit?
A. MMT abdominals, education of abdominal exercises to help restore abdominal strength
B. Education of pelvic floor exercises to help restore appropriate urinary frequency, body mechanics education
C. MMT pelvic floor, weight training exercises to help patient lift the baby and gear
D. ROMtests, electrical stimulation to help restore appropriate urinary frequency

A

B.
Reassurance and education are
key components to helping a new mother understand how to best take care of herself and her baby. Three weeks is too early to initiate an abdominal training program (or manually muscle test them), and usual recommendations following C-section include not lifting anything heavier than the baby during early recovery.

160
Q

In developing the plan of care for a 28-year-old pregnant woman, which of the following muscles should be the focus of the strengthening exercises to maintain a strong pelvic floor?
A. Piriformis, obturatorinternus, and pubococcygeus
B. Obturatorinterus, pubococcygeus, and coccyges
C. Rectus abdominis, iliococcy geus, and piriformis
D. Iliococcygeus, pubococcygeus, and coccygeus

A

D.
One of the main reasons that pelvic floor exercises are beneficial for a pregnant womanis the extra weight of the viscera.

161
Q

You ask your patient to extend her knee.
She lacks 10 degrees of reaching full extension.
In order for the knee to extend fully in this situation (sitting), the
A. Tibia must rotate medially on the femur
B. Tibia must rotate laterally on the femur
C. Femur must rotate medially on the tibia
D. Femur must rotate laterally on the tibia

A

B.
The tibia must rotate laterally
on the femur. Choice B describes the screw home mechanism in which the tibia rotates around the femur.

162
Q

Your 40-vear-old male patient was
playing basketball and describes hearing a pop and feeling as though someone kicked him in the back of the leg, followed by pain and weakness in the lower leg. The most common injury and special test to assess the problemis
A. Anterior cruciate ligament tear,
Lachman’s test
B. Achilles tendon rupture, Ober’s test
C. Anterior cruciate ligament tear, Thomas test
D. Achilles tendon rupture, Thompson test

A

D.
Achilles tendon rupture,
Thompson test is the correct choice. The injury mechanism described is a classic Achilles rupture. The Thompson test assesses the integrity of the Achilles tendon. The Thompson test involves the patient in the prone position.
The calf muscle complex is squeezed by the therapist. A positive test results in no movement of the ankle. If the ankle plantar flexes slightly, the Achilles tendon is considered intact.